A mysterious series

Let a1,a2,...a100a_1,a_2,...a_{100} be real numbers each less than 11,which satisfy, a1+a2+.....a100>1a_1+a_2+.....a_{100} > 1

1.1. Let n0n_0 be the smallest integer nn such that a1+a2+.....an>1a_1+a_2+.....a_n > 1 Show that the sums an0,an0+an01,......,an0+.....+a1a_{n_0},a_{n_0}+a_{n_0-1} ,......,a_{n_0}+.....+a_1 are positive

2.2.Show that there exists two integers pp and qq,p<qp < q,such that the numbers aq,aq+aq1,....,aq+.....+apa_q,a_q+a_{q-1},....,a_q+.....+a_p and ap,ap+ap+1,....,ap+.....+aqa_p,a_p+a_{p+1},....,a_p+.....+a_q

are all positive

#Sequences #Series #CosinesGroup #Goldbach'sConjurersGroup #TorqueGroup

Note by Eddie The Head
7 years, 2 months ago

No vote yet
1 vote

  Easy Math Editor

This discussion board is a place to discuss our Daily Challenges and the math and science related to those challenges. Explanations are more than just a solution — they should explain the steps and thinking strategies that you used to obtain the solution. Comments should further the discussion of math and science.

When posting on Brilliant:

  • Use the emojis to react to an explanation, whether you're congratulating a job well done , or just really confused .
  • Ask specific questions about the challenge or the steps in somebody's explanation. Well-posed questions can add a lot to the discussion, but posting "I don't understand!" doesn't help anyone.
  • Try to contribute something new to the discussion, whether it is an extension, generalization or other idea related to the challenge.
  • Stay on topic — we're all here to learn more about math and science, not to hear about your favorite get-rich-quick scheme or current world events.

MarkdownAppears as
*italics* or _italics_ italics
**bold** or __bold__ bold

- bulleted
- list

  • bulleted
  • list

1. numbered
2. list

  1. numbered
  2. list
Note: you must add a full line of space before and after lists for them to show up correctly
paragraph 1

paragraph 2

paragraph 1

paragraph 2

[example link](https://brilliant.org)example link
> This is a quote
This is a quote
    # I indented these lines
    # 4 spaces, and now they show
    # up as a code block.

    print "hello world"
# I indented these lines
# 4 spaces, and now they show
# up as a code block.

print "hello world"
MathAppears as
Remember to wrap math in \( ... \) or \[ ... \] to ensure proper formatting.
2 \times 3 2×3 2 \times 3
2^{34} 234 2^{34}
a_{i-1} ai1 a_{i-1}
\frac{2}{3} 23 \frac{2}{3}
\sqrt{2} 2 \sqrt{2}
\sum_{i=1}^3 i=13 \sum_{i=1}^3
\sin \theta sinθ \sin \theta
\boxed{123} 123 \boxed{123}

Comments

For 1., we prove by contradiction. Suppose one the sums, WLOG (an0++an0x) (a_{n_0} + \dots + a_{n_0 - x}) is not positive., i.e.(0) (\leq 0)

Now, As, a1+a2++an0>1 a_1 + a_2 + \dots + a_{n_0} > 1

(a1+a2++an0(x+1))+(an0x++an0)>1 \Rightarrow (a_1 + a_2 + \dots + a_{n_0 - (x+1)}) + ( a_{n_0 - x} + \dots + a_{n_0}) > 1

(a1+a2++an0(x+1))>1(an0x++an0) \Rightarrow (a_1 + a_2 + \dots + a_{n_0 - (x+1)}) > 1 - ( a_{n_0 - x} + \dots + a_{n_0})

(a1+a2++an0(x+1))>1 \Rightarrow (a_1 + a_2 + \dots + a_{n_0 - (x+1)}) > 1 (Since (an0x++an0) ( a_{n_0 - x} + \dots + a_{n_0}) is not positive)

But this contradicts the fact an0 a_{n_0} is the smallest n for which a1+a2++an>1 a_1 + a_2 + \dots + a_n > 1 .

Therefore our supposition is wrong and no sum is not positive, i.e. all the sums are positive

Siddhartha Srivastava - 7 years, 1 month ago
×

Problem Loading...

Note Loading...

Set Loading...